Search found 52 matches


Good call, ssmiles08. I thought it was D until I read what you said about K being 2 or 3. I now am 100% behind ssmiles08's answer.

by georgeung

Tue Aug 11, 2009 1:23 pm
Forum: Data Sufficiency
Topic: 20k
Replies: 4
Views: 1494

Oh man, this is a hard one... Is it A? Here's my reasoning, let me know if it's shotty. A: No way, there must be more days because the "Light" and "Moderate" are both lower. B: Can't be the same, they said 1990 was less. C: Can't be higher, they said 1990 was less. D: Total numbe...

by georgeung

Tue Aug 11, 2009 1:10 pm
Forum: Critical Reasoning
Topic: rainfall problem
Replies: 40
Views: 10861

That's tricky. I guessed C until I read the OA. All cows are lions and all lions are goats. all cows are goats. But that does not mean all goats are cows. If there are 100 cows, then there are 100 lions and then there are 100 goats. But if there's 103 goats, then only 100 cows are goats, but there a...

by georgeung

Tue Aug 11, 2009 12:45 pm
Forum: Data Sufficiency
Topic: cows, goats etc
Replies: 9
Views: 2212
by georgeung

Tue Aug 11, 2009 10:49 am
Forum: Critical Reasoning
Topic: Social systems
Replies: 7
Views: 2410

I think it's A.

Haha, three different people, three different answers.

by georgeung

Tue Aug 11, 2009 10:43 am
Forum: Critical Reasoning
Topic: Good health
Replies: 24
Views: 8648

"when they were children." is a subordinate clause. Because of the subordinate clause, "they" has to reference the young men and women again in order to have a subject. I can only see "they" referring to the young men and women. There's no other nouns "they" c...

by georgeung

Tue Aug 11, 2009 10:40 am
Forum: Sentence Correction
Topic: S-v doubt
Replies: 5
Views: 1095

I am going to say Answer B.

I counted in my head.

10 triangles using F.

5 triangles without F.

I'm interested in seeing how to solve this in a more efficient manner.

by georgeung

Tue Aug 11, 2009 10:06 am
Forum: Problem Solving
Topic: Interesting Geometry Question / Triangle
Replies: 19
Views: 3671

I picked C before I read the other posts and now I understand why D is the best answer. I know why to rule out A and B. The first does not support the overall position, but now I understand that the historians are not defending it either, which is why C is wrong. Scoobydooby explains it well why D s...

by georgeung

Tue Aug 11, 2009 8:59 am
Forum: Critical Reasoning
Topic: Newton
Replies: 5
Views: 1622

Since I don't know the OA, I'm going to assume it's A. This is a rough one, but even I have my doubts on A. A: Everything looks right with this, but the "had remained" part makes me question if it is right, but it seems to be since the simple past tense was used for 1989 so it makes sense ...

by georgeung

Tue Aug 11, 2009 8:33 am
Forum: Sentence Correction
Topic: one of only eight
Replies: 5
Views: 2373

I got the same thing.

Trees = 100

20%(100) = 20 Evergreens

80 trees left.
40%(100-20) = 32 Maple

75%(32) = 24 Oak

100-20-32-24= 24 other trees

by georgeung

Tue Aug 11, 2009 7:57 am
Forum: Problem Solving
Topic: Trees in a certain park
Replies: 7
Views: 8497

This is how I did the problem. Case A: 150/725 (real number) Case B: 2/3 (ratio) Combined: (150 + 2x) / (725 + 3x) = 3/5 The reason for the x is because case B is a ratio and we don't know what the real number is. Now we cross multiply: 3(720 + 3x) = 5(150 + 2x) 2175 + 9x = 750 + 10x 1425 = x 2x + 3...

by georgeung

Mon Aug 10, 2009 4:08 pm
Forum: Problem Solving
Topic: proportions q.
Replies: 5
Views: 2822

Data 1: I could only find two different answers for it - 2x3x5 and 5x6x1. So I put Insufficient. Data 2 - I found a couple, 7+3+0, 6+4+0 3+3+4 and 2+3+5. Enough answers for me to realize it is Insufficient. Then when I did both, I found 2+3+5 is the only one for Data 1 and Data 2 and the numbers cou...

by georgeung

Mon Aug 10, 2009 2:32 pm
Forum: Data Sufficiency
Topic: gmat prep 1
Replies: 2
Views: 1172

I think it is C. Which (referring to the plural "coolness of judgment and mutability of purpose...") were in fact natural outgrowths (plural) of it.

OA?

by georgeung

Mon Aug 10, 2009 1:46 pm
Forum: Sentence Correction
Topic: sc 1000
Replies: 3
Views: 3172

This is how I solved it: Class A Range = r Greatest Height = g Class B Range = s Greatest Height = h Data 1: r<s. The range for Class A is less than the range for class B. This is insufficient because we it doesn't tell us anything about the least height of the students. Data 2: g>h. The greatest he...

by georgeung

Mon Aug 10, 2009 12:15 pm
Forum: Data Sufficiency
Topic: Students
Replies: 1
Views: 1009